[Date Prev][Date Next][Thread Prev][Thread Next][Date Index][Thread Index]

Re: [obm-l] Problema



Title: Re: [obm-l] Problema
on 18.03.04 09:33, benedito at benedito@digizap.com.br wrote:

Problema
Um jogo entre duas pessoas, A  e  B, é definido da seguinte maneira:
A escolhe um intervalo fechado arbitrário J1, de comprimento menor do que 1;  
A seguir, B escolhe um intervalo fechado qualquer J2  contido em  J1, de comprimento menor do que 1/2.
Na sua vez de jogar, A escolhe um intervalo fechado qualquer contido em  J2, de comprimento  1/3; e assim por diante.
Pelo princípio dos intervalos encaixantes, a intereseção de  J1, J2, J3, ...  contém um único número real  r.
Se  r  for um número racional  A vence o jogo.
Se  r  for irracional, B vence o jogo.
Mostre que  A  tem uma estratégia vencedora, não importa com  B  jogue.

Oi, Benedito:

O enunciado parece dizer que, para n >= 3, o n-esimo intervalo escolhido tem que ter comprimento igual a 1/n, e nao menor do que 1/n. Eh isso mesmo?

[]s,
Claudio.